Vous êtes sur la page 1sur 25

Filière : Master 1 Probabilités et Statistiques

Projet

ANALYSE CONVEXE ET OPTIMISATION

Devoir de Maison

Sous la supervision du :
Prof Guy DEGLA
gdegla@imsp-uac.org

Présenté par :
NOM

NOM

NOM

ANNÉE UNIVERSITAIRE : 2022-2023

1
Exercice I
Exercice I
Déterminons parmi les problèmes d’optimisation donnés, ceux qui admettent de solutions puis justifions nos
réponses sans chercher à résoudre entièrement ces problèmes.
1. inf 2 (x2 + y 2 + e−y )
(x,y)∈R
∀(x, y) ∈ R2 , (x2 + y 2 + e−y ) ≥ 0.
Donc le problème admet de solution
2. inf (xy 2 )
(x,y)∈R2
Posons f (x, y) = xy 2
Supposons que f est minorée.
f est minorée, alors il existe un réel m tel que f (x, y) ≥ m∀x, y ∈ R
∀(x, y) ∈ R2 , f (x, y) ≥ m donc lim f (x, y) = −∞ ≥ m (Absurde). Donc f n’est pas minorée.
(x,y)→−∞
Donc le problème n’admet donc pas de solution.
3. inf (x2 − 3x + y)
x≥0,y≥0
x+y≤100
Posons D = {(x, y) ∈ R2 / x ≥ 0, y ≥ 0, x + y ≤ 100}
D est un ensemble compact. De plus, la fonction (x, y) 7→ x2 − 3x + y est continue sur R2 en particulier
sur D
Donc le problème admet de solution
4. sup (esin(x−y) )
(x,y)∈R2
Posons f (x, y) = esin(x−y)
∀x, y ∈ R − 1 ≤ sin(x − y) ≤ 1 donc e−1 ≤ f (x, y) ≤ e
Donc le problème admet de solution.
5. inf (x2 + y 2 − cos(x + y))
(x,y)∈R2
∀ (x, y) ∈ R2 −1 ≤ cos(x+y) ≤ 1 ; donc x2 +y 2 −1 ≤ x2 +y 2 −cos(x+y) ≤ x2 +y 2 +1 or x2 +y 2 −1 ≥ −1
donc x2 + y 2 − cos(x + y) ≥ −1.
Donc le problème admet de solution.
6. inf (cos2 x − y 2 )
x4 +y 4 ≤1
D = {(x, y) ∈ R2 , x4 + y 4 ≤ 1} est un compact. De plus la fonction (x; y) 7→ cos2 −y 2 est continue sur R,
en particulier sur D. D’où le problème admet de solution.
7. inf (x2 cos y)
(x,y)∈R2
Posons f (x, y) = x2 cos y
Supposons que f est minorée.
f est minorée alors il existe un réel m tel que f (x, y) ≥ m ∀x, y ∈ R. Donc ∀x ∈ R, f (x, π) = −x2 ≥ m,
donc lim f (x, π) = −∞ ≥ m (Absurde).
x→−∞
Donc le problème n’admet pas de solution
sin e(x−y)
8. sup 2+cos y
(x,y)∈R2
∀ (x, y) ∈ R2 − 1 ≤ sin e(x−y) ≤ 1 donc
−1 sin e(x−y) 1
2+cos y ≤ 2+cos y ≤ 2+cos y
De plus, −1 ≤ cos y ≤ 1 =⇒ 1 ≤ 2 + cos y ≤ 3 =⇒ 1
3 ≤ 1
2+cos y ≤1
(x−y)
Ainsi, sin e
≤1
2+cos y
Donc le problème admet de solution
R1
9. inf −1
[u(t)]2 dt
u∈C([0,1])
u(0)=1
u ∈ C([0, 1]), on a : [u(t)]2 ≥ 0
R1
alors −1 [u(t)]2 dt ≥ 0
donc le problème admet de solution.

2
Exercice II
1. i) Représentons graphiquement, dans différents repères, les épigraphes de chacune des fonctions définies
sur R par :
⋆ f (x) = x − 1

−2 −1 0 1 2 3 4

−1

−2

−3

Figure 1 – epi(f ) = {(x; t) ∈ R2 , f (x) ⩽ t}

⋆ g(x) = |x|

−3 −2 −1 0 1 2 3

−1

Figure 2 – epi(g) = {(x; t) ∈ R2 , g(x) ⩽ t}

⋆ h(x) = −|x|

3
2

−2 −1 0 1 2 3

−1

−2

−3

Figure 3 – epi(h) = {(x; t) ∈ R2 , h(x) ⩽ t}

ii) Parmi ces trois fonctions, celles qui sont convexes sont : f et g car leurs épigraphes sont des parties
convexe
2. Représentons graphiquement l’enveloppe convexe K des trois points suivants de R2 : A(0, 0), B = (0, 2)
et C = (1, 0).

B
2

A C

−2 −1 0 1 2

−1

−2

Écrire K comme l’ensemble des solutions d’un système de trois inéquations linéaires.

On a : (AB) : x = 0 (AC) : y = 0 (BC) : y = −2x + 2



x ⩾ 0

K comme l’ensemble des solutions d’un système de trois inéquations linéaires est : y ⩾ 0

2x + y ⩽ 2

4
Exercice III
1. i) Représentation des parties dans les figures différentes

−2 −1 0 1 2 3 4 5 6 7

−1

Figure 4 – K1 = {(x, y) ∈ R2 , x ⩾ 0 et y ⩾ 0}

−1 0 1 2 3 4 5

−1

Figure 5 – K2 = {(x, y) ∈ R2 , x > 0 et y > 0}

5
5

−6 −5 −4 −3 −2 −1 0 1 2 3 4 5 6

−1

−2

−3

−4

−5

Figure 6 – K3 = {(x, y) ∈ R2 , xy ⩾ 0}

−4 −3 −2 −1 0 1 2 3 4

−1

−2

−3

−4

Figure 7 – K4 = {(x, y) ∈ R2 , x + y ̸= 0}

6
4
2
(D)

−2 −1 0 1 2

−1

−2 0 2 4
Figure 9 – K6 = {(x, y) ∈ R2 , |x| = 1}

−2

Figure 8 – K5 = {(x, y) ∈ R2 , x − y = 1}

3 2

2
1
1

−3 −2 −1 0 1 2 3 −2 −1 0 1 2

−1
−1
−2

−3 −2

Figure 10 – K7 = {(x, y) ∈ R2 , |x| ⩽ 1 et |y| ⩽ 1}


Figure 11 – K8 = {(x, y) ∈ R2 , |x| + |y| = 1}

7
2
1

−2 −1 0 1 2

−2 −1 0 1 2
−1

−1
−2

Figure 12 – K9 = {(x, y) ∈ R2 , x2 + y 2 ⩽ 1} −2

Figure 13 – K10 = {(x, y) ∈ R2 , |x| + |y| ⩽ 1}

ii) Les ensembles convexes sont :


K1 ; K2 ; K5 ; K7 ; K9 ; K10 .

iii) Montrons par exploitation de la définition de la convexité d’un ensemble que :

a) K1 etK5 sont convexes

* Montrons
 que K12 est convexe :
K1 = (x; y) ∈ R /′ x ≥ 0 et y ≥ 0
x x
soit A et B ′ ∈ K1 et t ∈ [0; 1]
y y
Notons par C=tA+(1 − t)B.
Etmontrons queC ∈K1
tx + (1 − t)x′
C
ty + (1 − t)y ′
A ∈ K1 et B ∈ K1 =⇒ x ≥ 0 ; x′ ≥ 0 y ≥ 0 et y ′ ≥ 0
Donc :
c’est mieux de mettre Donc au lieu de en suite ∀t ∈ [0; 1], tx + (1 − t)x′ ≥ 0 et
ty + (1 − t)y ′ ≥ 0 car t≥ 0 et (1 − t) ≥ 0 ; il s’ en suit que C ∈ K1
On conclu que K1 est convexe

** Montrons
 que K52 est convexe :
K5 = (x; y) ∈ R /x − y = 1
x x′
soit A et B ′ ∈ K5 et t ∈ [0; 1]
y y
Notons par C=tA+(1 − t)B.
Etmontrons queC ∈ K5
tx + (1 − t)x′
C
ty + (1 − t)y ′
A; B ∈ K5 =⇒ x − y = 1 et x′ − y ′ = 1.
On a : ∀ t ∈ [0; 1]

8
tx + (1 − t)x′ − [ty − (1 − t)y ′ ] = tx + (1 − t)x′ − ty − (1 − t)y ′
= t(x − y) + (1 − t)(x′ − y ′ )
= t+1−t
= 1
Par suite, C ∈ K5 .
On conclut que K5 est convexe.

b) Montrons
 que K6 n’est pas convexe.
K6 = (x; y) ∈ R2 /|x| = 1
   
1 −1 1
soit A ,B et t= .
1 2 2
On a bien A ; B ∈ K6 et t ∈ [0; 1] Mais. |t(1) + (1 − t)(1)| = |− 12 + 1 − 12 | = 0 ̸= 1
Donc tA+(1 − t)B ∈/ K6 .
On conclut que K6 n’ est pas convexe.
2. Considérons les ensembles :
{P ∈ R [X] ; P (0) = −1}
E1 = n o
R1
E2 = f ∈ C([0, 1]) : 0 |f (x)|dx ≤ 1
A = {f ∈ C([0, 1]) : |f (0)dx = 1}

i Montrons que E1 est convexe. :


soit P1 et P2 ∈ E1 et t ∈[0, 1]
P1 , P2 ∈ E1 ⇔ P1 (0) = −1 etP2 (0) = −1
Montrons que tP1 +(1-t)P2 ∈ E1
On a :

tP1 +(1-t)P2 ∈ R[X] car R[X] est un espace vectoriel et

tP1 (0) + (1 − t)P2 (0) = (−1)t + (−1)(1 − t)


= −t − 1 + t
= −1

tP1 + (1 − t)P2 ∈ E1 donc E1 est alors convexe

ii Montrons que E2 est convexe


soit f et g ∈ E2 et t∈ [0, 1]
R1
f∈ E2 ⇐⇒ f ∈ C([0, 1]) et 0 |f (x)|dx ≤ 1
R1
g∈ E2 ⇐⇒ g ∈ C([0, 1]) et 0 |g(x)|dx ≤ 1
donc
tf+(1-t)g∈ C([0, 1])
on sait que :|(tf + (1 − t)g)(x)| ≤ |(tf )(x)| + |(1 − t)g(x)| d’après l’inégalité triangulaire.
Il s’en suit que :
R1 R1
0
|(tf + (1 − t)g)(x)|dx ≤ 0
(|tf (x)| + |(1 − t)g(x)|)dx
R1 R1
≤ 0
|(tf )(x)|dx + 0
|(1 − t)g(x)|dx
R1 R1
≤ 0
t|f (x)|dx + 0
(1 − t)|g(x)|dx (car t ≥ 0 et (1 − t) ≥ 0)
R1 R1
≤ t 0
|f (x)|dx + (1 − t) 0
|g(x)|dx

≤ t+1−t

≤ 1
Donc tf+(1-t)g ∈E2 .
D’où E2 est convexe.

9
Exercice IV
On considère la programmation linéaire :

(Lp1 ) max (x+4y)


x ≥ 0, y ≥ 0
2xy ≤ 12, x + 2y ≤ 16
x − 2y + 12 ≥ 0
avec : f (x, y) = x + 4y ∀(x, y) ∈ IR et
K = {(x; y) ∈ IR \ x ≥ 0, y ≥ 0, 2x y ≤ 12, x + 2y ≤ 16, x − 2y + 12 ≥ 0}

1. Dessinons le Polygone K.
Soit (D1 ), (D2 ), (D3 ), (D4 ), et (D5 ) les droites d’équation respective :
(D1 ) : x = 0
(D2 ) : y = 0
(D3 ) : 2x − y = 12
(D4 ) : x + 2y = 16
(D5 ) : x − 2y + 12 = 0
Les points A(6 ;0) et B(4 ;-4) sont deux points de (D3 )
Les points C(6 ;5) et D(4 ;6) sont deux points de (D4 )
Les points E(0 ;6) et F(-4 ;4) sont deux points de (D5 )
On a :

10
13

12

11

10

(D4 ) 9
(D5 )

8
A2
7
A1
6 (D3 )

A3
4

(∆6 )
2

A4
A0
−6 −5 −4 −3 −2 −1 0 1 2 3 4 5 6 7 8 9 10

−1

(∆1 )
−2

−3

Conclusion
La représentation du polygone K est la partie non colorée.

2. Résolvons graphiquement le problème (Lp1 )


1<6 et d’après la représentation graphique, la ligne de niveau (∆1 ) est en dessous de la ligne de niveau
(∆6 ) donc le problème (Lp1 ) admet de solution. Cette solution est atteint au point A2 (2; 7) et vaut
f (2; 7) = 30
3. i) Trouvons les points représentant les sommets de K
Les points A0 (0; 0), A1 (0; 6), A2 (2; 7), A3 (8; 4) et A4 (6; 0) sont les sommets de K
ii) Retrouvons la solution de (Lp1 ) par la méthode d’énumération.

Les points Ai A0 A1 A2 A3 A4
f (Ai ) 0 24 30 24 6
D’après le tableau précédent, la solution du problème (Lp 1) est atteint au point A2 (2; 7) et
vaut f (A2 ) = 30

11
Exercice V
On considère la programmation linéaire :

(Lp1 ) : max (x+2y)


x ≥ 0, y ≥ 0
3x + 2y ≤ 12
−x + 2y ≤ 4
Posons : f (x, y) = x + 2y ∀(x, y) ∈ IR et
G = {(x; y) ∈ IR \ x ≥ 0, y ≥ 0, 3x + 2y ≤ 12 et − x + 2y ≤ 4}

Résolvons le problème (Lp1 ) :

1. Graphiquement.
Soit (D1 ), (D2 ), (D3 ), (D4 ), et (D5 ) les droites d’équation respective :
(D1 ) : x = 0
(D2 ) : y = 0
(D3 ) : 3x + 2y = 12
(D4 ) : −x + 2y = 4
Les points A(4 ;0) et B(2 ;3) sont deux points de (D3 )
Les points C(0 ;2) et D(-4 ;0) sont deux points de (D4 )
La représentation graphique de G est donné par :

5 (D3 )

4 (D4 )

(∆4 ) A2
3

A1
2

A0 A3
−4 −3 −2 −1 0 1 2 3 4 5 6 7

−1
(∆2 )

−2

La représentation du domaine G est la partie non colorée.

2<4 et d’après la représentation graphique de G, la ligne de niveau (∆2 ) est en dessous de la ligne de
niveau (∆4 ) donc le problème (Lp1 ) admet de solution. Cette solution est atteinte au point A2 (2; 3)
et vaut f (2; 3) = 8
2. Résolvons le problème (Lp1 ) en utilisant les variables d’écart
Posons : t = −3x − 2y + 12 et s = x − 2y + 4

12
t−s+8 −s − 3t + 24
On a donc s ≥ 0 ; t ≥ 0 ; x = et y =
4 8
Soit h la fonction de IR vers IR tel que : f (x; y) = h(s; t). On a donc :

f (x; y) = h(s; t)
 
t−s+8 −s − 3t + 24
= +2
4 8

−4s − 4t + 64
=
8
(s + t)
= − +8
2
(s + t)
s ≥ 0 et t ≥ 0 donc − ≤ 0. Ainsi les valeurs de s et t pour lesquelles f est maximale sont :
2
s = 0 et t = 0.
s = 0 et t = 0 ⇒ x = 2 et y = 3. Ainsi la solution du problème (Lp1 ) est obtenue au point de coordonné
(2 ;3) et vaut f (2; 3) = 8.

Exercice VI
1°) Vérifions :

i°) f : R → R, f (x) = -x+1


f est convexe car f est affine.

ii°) f : R → R, f (x) = |x|


Soit t ∈[0,1], ∀ (u,v)∈R², on a

f (tu + (1 − t)v) = |tu + (1 − t)v|


≤ |tu| + |(1 − t)v|
≤ t|u| + (1 − t)|v|
f (tu + (1 − t)v) = tf (u) + (1 − t)f (v)

D’où f est convexe. iii°) f : R → R, f (x) = −x2


f n’est pas convexe car ∀x ∈ R, f ′′ (x) = −2 < 0..

iv°) f : R → R, f (x) = −|x|


f n’est pas convexe car la fonction x → |x| est convexe.

v°) f : R → R, f (x) = (−x + 1)2


f est convexe car f ′′ (x) = 2 > 0, ∀x ∈ R.

vi°) f : R → R, f (x) = x3
f n’est pas convexe car ∀x ∈ R∗− , f ′′ (x) = 6x < 0.

vii°) f : R+ → R, f (x) = x3
f est convexe car ∀x ∈ R+ , f ′′ (x) = 6x ≥ 0.

viii°) f 
: R → R, f (x) = max {1 − x, 0}
1−x si x<1
f (x) = .
0 sinon
Or les fonctions x → 1 − x et x → 0 sont convexes sur R.
Donc f est convexe.

ix°) f : R → R, f (x) = ex
f est convexe car f ′′ (x) = ex > 0, ∀x ∈ R.

13
x°) f : R → R, f (x) = e−x
f est convexe car f ′′ (x) = e−x > 0, ∀x ∈ R.

xi°) f : R → R, f (x) = x4 − x3 + x2 − 1
f est au moins deux fois dérivable sur R et on a :
f ′′ (x) = 12x2 − 6x + 2
Donc ∀ x ∈ R, f est convexe
∆ = −60 < 0
xii°)f : R → R, f (x) = x4 − x2
f est au moins deux fois dérivable sur R et on a f ′′ (x) = 12x − 2 et f ′′ (0) = −2 < 0. D’où f n’est pas convexe.

2°) Vérifions :

i°) g : R2 → R, g(x, y) = −x + y − 1
g est convexe car affine.

ii°) g : R2 → R, g(x, y) = |x| − y


g est convexe car c’est la somme de deux fonctions convexes : (x; y) 7→ |x| et (x; y) 7→ −y

iii°) g : R2 → R, g(x, y) = −y 2
g n’est pas convexe car la fonction (x, y) → y 2 est convexe.

iv°) g : R2 → R, g(x, y) = x2 + (y + 1)4


Soit Hg la matrice hessienne
 de la fonction g.
2 0
Hg =
0 12(y + 1)2
tr(Hg ) = 2 + 12(y + 1)2 > 0 et det(Hg ) = 24(y + 1)2 ≥ 0

v°)g : R2 → R, g(x, y) = ex + e−y


g est convexe car elle est somme de deux fonctions convexes : (x; y) 7→ ex et (x; y) 7→ e−y

vi°)g : R2 → R, g(x, y) = xy
Soit Hg la matrice
 hessienne de la fonction g.
0 1
Hg =
1 0
tr(Hg ) = 0 et det(Hg ) = −1 < 0, donc Hg n’est pas définie semie-définie positive. D’où g n’est pas convexe.

vii°) g : R2 → R, g(x, y) = x2 + xy + y 2
Soit Hg la matrice
 hessienne de la fonction g.
2 1
Hg =
1 2
tr(Hg ) = 4 et det(Hg ) = 3, donc Hg est définie semie-positive. D’où g est convexe.

viii°) g : R2 → R, g(x, y) = x2 − y 2
Soit Hg la matrice
 hessienne de la fonction g.
2 0
Hg =
0 −2
tr(Hg ) = 0 et det(Hg ) = −4, donc Hg n’est pas semie-positive. D’où g n’est convexe.

3°) i°) Soitn ∈ N∗ , fn : R 7→ R, fn (x) = xn , ∀x ∈ R.

Trouvons toutes les valeurs de n :

* Pour n = 1, ∀x ∈ R, f1 (x) = x.
La fonction x 7→ x est affine donc f1 est convexe.

14
* Pour n = 2, ∀x ∈ R, f2 (x) = x2 .
La fonction x 7→ x2 est convexe donc f2 est convexe.
Soit n > 2
La fonction x 7→ xn est au moins deux fois dérivable sur R et on a : fn′′ (x) = n(n − 1)xn−2 .
* Pour n − 2 pair
n − 2 pair ⇒ n pair et ∀x ∈ R, fn′′ (x) ≥ 0
donc fn est convexe.
* Pour n − 2 impair
n − 2 impair ⇒ n pair. On a :
∀x ∈ R∗− , fn′′ (x) < 0
∀x ∈ R+ , fn′′ (x) ≥ 0
donc fn est convexe.
En conclusion, fn est convexe si et seulement si n ∈ {1} ∪ {2k, k ∈ N∗ }.

ii°) Soitp ∈ R∗+ , fp : R+ 7→ R, fp (x) = xp , ∀x ∈ R.

Trouvons toutes les valeurs de p :

* Pour p ∈]2; +∞[.


∀x ∈ R+ , la fonction x 7→ xp est au moins deux fois dérivable et on a : fp′′ (x) = p(p − 1)xp−2 et
∀x ∈ R+ , fp′′ (x) ≥ 0
donc fp est convexe.
* Pour p ∈]0; 2[, fp est dérivable sur R∗+ et ∀x ∈ R+ , on a : fp′′ (x) = p(p − 1)xp−2 ≥ 0.
f ′′ (x)p ≥ 0 ⇔ p(p − 1)xp−2 ≥ 0
⇔ (p − 1) ≥ 0
⇔ p≥1
* Pour p ∈]0; 1[, on a : fp′′ (x) ≤ 0 alors fp n’est pas convexe sur R∗+ . Donc elle n’est pas convexe sur
R+ car R∗+ ⊂ R+
* Pour p ∈]1; 2[, on a : fp′′ (x) ≥ 0, ∀x ∈ R∗+ alors fp est convexe sur R∗+ .
Soient x et y ∈ R+
x, y ∈ R+ ⇔ x=0 et y=0
ou x=0 et y ∈ R∗+
Montrons que ∀t ∈]0; 1[, On a fp (tx+(1−t)y) ≤ tf (x)+(1−t)f (y). On a
ou y ∈ R∗+ et x=0
ou x, y ∈ R∗+
* Pour x = 0 et y = 0
fp (tx + (1 − t)y) ≤ tf (x) + (1 − t)f (y)
* Pour x = 0 et y ∈ R∗+
fp (tx + (1 − t)y) = (1 − t)p y p
fp (x) = 0
fp (y) = y p
(1 − t) ∈ [0; 1] alors (1 − t)p ≤ (1 − t)p ∈]1; 2[
(1 − t)p ≤ (1 − t) ⇒ (1 − t)p y p ≤ (1 − t)y p
⇒ fp (tx + (1 − t)y) ≤ (1 − t)fp (y) Donc pour x = 0 et y ∈ R∗+ On a :
⇒ fp (tx + (1 − t)y) ≤ tfp (x) + (1 − t)fp (y)
fp (tx + (1 − t)y) ≤ tfp (x) + (1 − t)fp (y)
* Par analogie, pour x ∈ R∗+ ety = 0
* pour x ∈ R∗+ et y ∈ R∗+ , On a : fp (tx + (1 − t)y) ≤ tfp (x) + (1 − t)fp (y) car fp est convexe sur R∗+ .
∀x, y ∈ R+ , On a :
fp (tx + (1 − t)y) ≤ tfp (x) + (1 − t)fp (y).
Donc pour p ∈]1; 2[, fp est convexe sur R+ .
De tout ce qui précède on retient que f est convexe si et seulement si p ∈ [1; +∞]

15
Exercice VII
Soit Q le polynôme défini sur R2 par Q(x; y) = ax2 + by 2 + cx + dy + e ; où a, b, c, d et e sont des paramètres
réels.

Cherchons les valeurs des paramètres pour lesquelles :

1. Q est affine :
Q est affine si et seulement si a = 0, b = 0, c, d, et e ∈ R.
2. Q est quadratique :
Q est quadratique si et seulement si (a; b) ̸= (0; 0) et (c; d; e) = (0; 0; 0).
3. Q est convexe :
Soit H
 la matrice hessienne de la fonction Q.
2a 0
H=
0 2b
Q est convexe ⇔ les valeurs propres soient positives
⇔ 2a ≥ 0 et 2b ≥ 0
⇔ a ≥ 0 et b ≥ 0
En conclusion Q est convexe si et seulement si a ≥ 0, b ≥ 0, c, d et e ∈ R
4. Q est coercif si et seulement si (a; b) ̸= (0; 0) et (c; d; e) ∈ R3 .

Exercice VIII
Soit K ⊂ Rn un ensemble non vide et fermé.
Justifions que pour tout élément x0 de Rn , il existe un unique élément y0 ∈ K tel que
inf ∥y − x0 ∥2 =∥y0 − x0 ∥2
Soit x0 un élément de Rn et f :K → R une fonction continue définie par f(y)= ∥y − x0 ∥2 .

EXISTENCE

Cas 1 : L’ensemble K est borné.


K étant borné et fermé de Rn alors K est compact.Comme f est continue , d’après le théorème de Weierstrass
f est bornée sur K et elle atteint ses bornes. Donc il existe au moins un point minimum de f sur K

Cas 2 : L’ensemble K est non borné


Pour tout y ∈ K,
(
lim∥y∥→+∞ f (x) = lim∥y∥→+∞ ∥y − x0 ∥2
(1)
= +∞ car ∥y − x0 ∥2 ≥ ∥x∥2 − ∥x0 ∥2
donc f est coercive sur un sous-ensemble non vide et non borné d’un espace normé Rn de dimension finie,d’après
Weierstrass ,f admet un minimum

UNICITE
Soit y1 , y2 deux éléments de K tel que :

δ = d(y, K) = ∥y1 − x0 ∥2 = ∥y2 − x0 ∥2 (2)

La norme euclidienne vérifie la loi du parallèlogramme.


En prenant x=x0 − y1 et = x0 − y2
∥y1 − y2 ∥22 = ∥(x0 − y1 ) − (x0 − y2 )∥22
= 2(∥x0 − y1 ∥22 + ∥x0 − y2 ∥22 ) − (∥x0 − y1 + x0 − y2 ∥2 )2
= 4δ 2 − 4∥x0 − y1 +y 2 2 (3)
2 ∥2
y 1 + y2 2
= 4(δ 2 − ∥x0 − ∥2 )
2
0 ≤ ∥y1 − y2 ∥22 ⇔ ∥y1 − y2 ∥2 = 0, alorsy1 = y2

16
Exercice IX
1. i) La condition suffisante pour que les sous-ensembles {f ≤ γ} et {f < γ} soient convexes est que la
fonction f soit convexe.
En effet, supposons que f est convexe.
Soient x, y ∈ {f ≤ γ} et λ ∈ [0, 1]. Cela signifie que f (x) ≤ γ, f (y) ≤ γ. Nous voulons montrer que
λx + (1 − λ)y ∈ {f ≤ γ}. Comme f est convexe, nous avons f (λx + (1 − λ)y) ≤ λf (x) + (1 − λ)f (y) ≤
λγ + (1 − λ)γ = γ, donc λx + (1 − λ)y ∈ {f ≤ γ}.
De même, nous pouvons montrer que si f est convexe, alors {f < γ} est également convexe.

Soient w, z ∈ {f < γ} et λ ∈ [0, 1]. Cela signifie que f (w) < γ, f (z) < γ. Nous voulons montrer que
λw + (1 − λ)z ∈ {f < γ}. Comme f est convexe, nous avons f (λw + (1 − λ)z) < λf (w) + (1 − λ)f (z) <
λγ + (1 − λ)γ = γ, donc λw + (1 − λ)z ∈ {f < γ}.Alors {f < γ} est également convexe.

En somme la convexité de f une condition suffisante pour la convexité des ensembles {f ≤ γ} et


{f < γ}

ii) L’ensemble {x ∈ R : x3 − 1 ≤ 0} est-il connexe ?


{x ∈ R : x3 − 1 ≤ 0} ={x ∈ R : x3 ≤ 1}
Posons f (x) = x3
Vérifions si f est convexe
f est continue sur R et est deux fois différentiables alors : f ′ (x) = 3x2
f ′′ (x) = 6x, qui n’est pas toujours positive donc f n’est pas convexe.

Verifions si les ensembles sont convexes

Soit x,y ∈ {x ∈ R : x3 ≤ 1} et λ ∈ [0, 1] , alors f (x) ≤ 1,f (y) ≤ 1 ,on a :

  3

f (x) ≤ 1 
 x ≤1

f (y) ≤ 1 y 3 ≤ 1

=⇒


0<λ<1 

 0<λ<1
0<1−λ<1 0<1−λ<1
 


 x≤1

y ≤ 1
=⇒


 0<λ<1
0<1−λ<1

(
λx ≤ x
=⇒
(1 − λ)y ≤ 1 − λ
n
=⇒ λ + (1 − λ)y ≤ 1
n
=⇒ (λ + (1 − λ)y)3 ≤ 13
n
=⇒ f (λ + (1 − λ)y) ≤ 1
n
=⇒ λ + (1 − λ)y ∈ {f ≤ 1}

Alors {f ≤ 1} est connexe.

L’ensemble {x ∈ R : x3 − 1 < 0} est-il connexe ?


{x ∈ R : x3 − 1 < 0} ={x ∈ R : x3 < 1}
Posons f (x) = x3
Soit x,y ∈ {x ∈ R : x3 < 1} et λ ∈ [0, 1] , alors f (x) ≤ 1,f (y) ≤ 1 ,on a :

17
  3

 f (x) < 1 
 x <1

f (y) < 1 y 3 < 1

=⇒


 0<λ<1 

 0<λ<1
0<1−λ<1 0<1−λ<1
 


 x<1

y < 1
=⇒


 0<λ<1
0<1−λ<1

(
λx < x
=⇒
(1 − λ)y < 1 − λ
n
=⇒ λ + (1 − λ)y < 1
n
=⇒ (λ + (1 − λ)y)3 < 13
n
=⇒ f (λ + (1 − λ)y) < 1
n
=⇒ λ + (1 − λ)y ∈ {f < 1}

Alors {f < 1} est connexe.


Conclusion :La convéxité de f est une condition suffisante mais pas nécessaire pour que les ensembles
{x ∈ R : x3 − 1 ≤ 0} et {x ∈ R : x3 − 1 < 0} soient convexes
2. i) Une condition suffisante pour que le sous-ensemble f = γ = f −1 (γ) = x ∈ K : f (x) = γ soit convexe
est que la fonction f soit affine sur tout segment reliant deux points de cet ensemble. Autrement dit,
si f est une fonction affine, alors le sous-ensemble f = γ est convexe pour tout réel γ.
ii) L’ensemble {x ∈ R : x3 − 1 = 0} est convexe,car {x ∈ R : x3 − 1 = 0}={x ∈ R : x = 1}
C’est la droite d’équation x=1, d’ou {x ∈ R : x3 − 1 = 0} est convexe.
Or en posant f (x) = x3 ,f n’est pas convexe

La leçon à en tirer :la convexité de f est une condition suffisante mais pas nécessaire pour la convexité
de l’ensemble {f = γ}
3. {f ≥ γ} ={−f ≤ −γ}
{f > γ}={−f < −γ}
{f ≥ γ} et {f > γ} sont convexes si (-f) est convexe donc f est concave d’après la réponse à la question
1-i)

Exercice X
On veut implanter la station d’une télévision numérique dans une région plane rapportée à un repère ortho-
normé en un point P (x, y) tel que la somme f (x, y) des carrés des distances de P aux villes A(0, 0), B(1, 3) et
C(5, 0) soit minimum.
1. Justifions que f (x, y) = x2 + (x − 1)2 + (x − 5)2 + 2y 2 + (y − 3)2

18
4

B
3

P
1

A C

−1 0 1 2 3 4 5 6

−1

Soit P (x, y)

f (x, y) = AP 2 + BP 2 + CP 2
= x2 + y 2 + (x − 1)2 + (y − 3)2 + (x − 5)2 + y 2
= x2 + (x − 1)2 + (x − 5)2 + 2y 2 + (y − 3)2

Donc f (x, y) = x2 + (x − 1)2 + (x − 5)2 + 2y 2 + (y − 3)2


2. Trouvons les coordonnées du point P qui minimise f .

Soit P (s, y)

f (x, y) = x2 + (x − 1)2 + (x − 5)2 + 2y 2 + (y − 3)2


3x2 − 12x + 3y 2 − 6y + 35
=
   
6x − 12 6 0
On a : ∇f (x, y) = et Hessf (x, y) =
6x − 6 ( 0( 6
   
6x − 12 0 6x − 12 = 0 x=2
∇f (x, y) = = ⇔ ⇔
6x − 6 0 6x − 6 = 0 y=1
De plus la matrice hessienne de f est semi-définie positive.
Par suite le minimum est atteint au point (2, 1)

D’oùP (2, 1)
.

Exercice XI
Soit n ⩾ 1 un entier naturel, A une matrice réelle symétrique d’ordre n, Q une matrice réelle carrée et
d’ordre n, b ∈ Rn un vecteur et c ∈ R une constante scalaire. On pose pour tout x ∈ Rn ,
1
f (x) = < Ax, x > + < b, x > +c
2
1
g(x) = < Qx, x >
2
φ(x) = ∥x − b∥2

1. Déterminons en un point quelconque de x ∈ Rn ,


i- le gradient de f .
1
f (x) = < Ax, x > + < b, x > +c
2

19
f est différentiable sur Rn . Soit h ∈ Rn

f (x + h) = f (x) + ∇f (x)hT + o(∥h∥)

1
f (x + h) = < A(x + h), x + h > + < b, x + h > +c
2
1 1 1 1
= < Ax, x > + < Ax, h > + < Ah, x > + < Ah, h > + < b, x > + < b, h > +c
2 2 2 2
1 1 1
= f (x) + < Ax, h > + < h, Ax > + < Ah, h > + < b, h >
2 2 2
1 1 T 1
= f (x) + < Ax, h > + < A x, h > + < Ah, h > + < b, h >
2 2 2
1
= f (x)+ < Ax, h > + < b, h > + < Ah, h >
2
car A = AT
1
f (x + h) = f (x)+ < Ax + b, h > + < Ah, h >
2

On a :
< Ah, h >
⩽ ∥Ah∥
∥h∥
Or ∥Ah∥ → 0 quand ∥h∥ → 0, donc ∥ < Ah, h > ∥ → 0 quand ∥h∥ → 0
Par conséquent,
∇f (x) = Ax + b
ii- Le gradient de g

En prenant b = c = 0 et A = Q on a : f (x) = g(x)


D’où
∇g(x) = Qx
.
iii- Le gradient de φ

φ(x) = ∥x − b∥2

φ est différentiable sur Rn .

φ(x + h) = ∥x + h − b∥2
=< x + h − b, x + h − b >
= φ(x) + 2 < x − b, h > +∥h∥2

Or ∥h∥2 −→ 0 quand ∥h∥ −→ 0, donc

∇φ(x) = 2(x − b)

2. i- Matrice hessienne de f
f est de classe C 2 sur Rn .
On a :
1
f (x + h) = f (x) + ∇f (x)hT + Hess(f )h + o(∥h∥2 )
2
Or
1
f (x + h) = f (x)+ < Ax + b, h > + < Ah, h >
2
Donc
Hess(f ) = A

20
ii- Condition suffisante et nécessaire
f est convexe ⇐⇒ Hess(f ) est définie semi-positive
Donc,
f est convexe ⇐⇒ A est définie semi-positive, b ∈ Rn , c ∈ R

Exercice XII
1. Nous cherchons à minimiser la fonction f (x, y, z) = x2 + y 2 + z 2 sous la contrainte g(x, y, z) = x + y +
z − 1 = 0. Nous pouvons appliquer la méthode des multiplicateurs de Lagrange en considérant la fonction
Lagrangienne L(x, y, z, λ) = f (x, y, z) + λg(x, y, z) = x2 + y 2 + z 2 + λ(x + y + z − 1).
En résolvant les équations du système suivant :

∂L
= 2x + λ = 0
∂x
∂L
= 2y + λ = 0
∂y
∂L
= 2z + λ = 0
∂z
∂L
=x+y+z−1=0
∂λ

Nous obtenons λ = −2x = −2y = −2z et x+y +z = 1. En utilisant la contrainte, nous avons z = 1−x−y,
ce qui donne λ = 2x + 2y − 2.
En éliminant λ entre les deux premières équations, nous obtenons x = y. En éliminant λ entre la première
équation et la contrainte, nous avons x = 13 . Ainsi, y = 13 et z = 13 . Par conséquent, le minimum est atteint
pour x = 31 , y = 13 et z = 13 , avec une valeur de f (x, y, z) = 13 .

2. On cherche à minimiser la fonction f (x, y, z) = x + y + z sous la contrainte g(x, y, z) = x2 + y 2 + z 2 − 1 = 0.


On peut utiliser la méthode des multiplicateurs de Lagrange en définissant la fonction Lagrangienne :
L(x, y, z, λ) = f (x, y, z) + λg(x, y, z) = x + y + z + λ(x2 + y 2 + z 2 − 1)
Les conditions du premier ordre pour un minimum sont données par :
∂L
= 1 + 2λx = 0
∂x
∂L
= 1 + 2λy = 0
∂y
∂L
= 1 + 2λz = 0
∂z
∂L
= x2 + y 2 + z 2 − 1 = 0
∂λ
En résolvant le système d’équations, on obtient :
1 1
x = y = z = ±√ , λ=− √
3 2 3
La fonction f (x, y, z) = x + y + z est strictement croissante le long des rayons partant de l’origine, donc le
minimum de f sous la contrainte g(x, y, z) = x2 + y 2 + z 2 − 1 = 0 est atteint pour (x, y, z) = ( √13 , √13 , √13 ),

avec une valeur minimale de f ( √13 , √13 , √13 ) = 3.

21
3. On introduit le multiplicateur de Lagrange λ ∈ R et on considère la fonction :

L(x, y, z, λ) = 2xy + y 2 + z 2 + λ(x + y − 1)

On cherche les points stationnaires de cette fonction en cherchant les points où les dérivées partielles sont
nulles :  ∂L

 ∂x = 2y + λ = 0
 ∂L = 2x + 2y + λ = 0

∂y
∂L
∂z = 2z = 0



 ∂L
∂λ = x + y − 1 = 0
La troisième équation donne z = 0, et la quatrième donne y = 1 − x. En substituant cela dans les deux
premières équations, on obtient :

1
x = − 2
( 
2(1 − x) + λ = 0
=⇒ y = 32
2x + 2(1 − x) + λ = 0 
λ=2

On vérifie facilement que ce point est un minimum global de la fonction en utilisant la méthode de la
dérivée seconde : la matrice hessienne de L est donnée par :
 
0 2 0 1
 2 2 0 1
HL =  0 0 2 0

1 1 0 0

En évaluant cette matrice en (x, y, z, λ) = (− 12 , 23 , 0, 2), on obtient :


   
0 2 0 1 0 2 0 1
2 2 0 1 2 2 0 1
HL =    = 
0 0 2 0 0 0 2 0
1 1 0 0 (− 1 , 3 ,0,2) 1 1 0 0
2 2

Le déterminant de cette matrice est non nul, donc elle est inversible, et les sous-matrices principales sont
également non nulles, donc cette matrice est définie positive. Par conséquent, le point trouvé est bien un
minimum global de la fonction sur le domaine de contrainte. Le minimum est atteint en (x, y, z) = ( 12 , 21 , 0)
avec une valeur de 1/2.

Exercice XIII
Utilisons la méthode de Karush-Kuhn-Tucker pour résoudre les problèmes d’optimisation suivants :

a) Minimiser f (x, y, z) = x2 − 2x + y 2 + z 2 sur R3 sous les contraintes h1 (x, y, z) = x + y − 1 = 0 et h2 (x, y, z) =


x − 2 ≥ 0.
Les conditions de stationnarité, de faisabilité et de complémentarité donnent :

2x − 2 + λ1 + µ = 0,
2y + λ1 = 0,
2z + λ2 = 0,
y + x − 1 = 0,
x − 2 ≥ 0,
µ ≥ 0,
λ1 ≥ 0,
λ2 ≥ 0,
λ1 (y + x − 1) = 0,
µ(x − 2) = 0.

22
Il y a trois cas à considérer :
*Cas 1 : µ > 0, λ1 = 0, λ2 = 0**
Dans ce cas, les équations de KKT deviennent :

2x − 2 + µ = 0,
2y = 0,
2z = 0,
y + x − 1 = 0,
x − 2 ≥ 0,
µ > 0.
En résolvant ces équations, on trouve x = 1/2, y = 1/2, z = 0 et µ = 3. Cette solution satisfait également la
contrainte x − 2 ≥ 0, donc elle est réalisable. Ainsi, nous avons une solution réalisable optimale avec x = 1/2,
y = 1/2 et z = 0, et une valeur de la fonction objectif de f (1/2, 1/2, 0) = 1/2.
*Cas 2 : µ = 0, λ1 > 0, λ2 = 0**
Dans ce cas, les équations de KKT deviennent :

2x − 2 + λ1 = 0,
2y + λ1 = 0,
2z = 0,
y + x − 1 = 0,
x − 2 ≥ 0,
λ1 > 0.
En résolvant ces équations, on trouve x = 3/2, y = −1/2 et λ1 = 2. Cette solution ne satisfait pas la
condition de faisabilité, donc elle n’est pas une solution optimale.
*Cas 3 : µ = 0, λ1 = 0, λ2 > 0**
Dans ce cas, les équations de KKT deviennent :

2x − 2 = 0,
2y = 0,
2z + λ2 = 0,
y + x − 1 = 0,
x − 2 ≥ 0,
λ2 > 0.
En résolvant ces équations, on trouve x = 1, y = 0 et λ2 = 1. Cette solution ne satisfait pas la condition de
faisabilité, donc elle n’est pas une solution optimale.
Ainsi, la solution optimale est x = 1/2, y = 1/2 et z = 0, avec une valeur de la fonction objectif de 1/2.

b) La fonction objectif est convexe et les contraintes sont des inégalités.


Les conditions KKT sont les suivantes :

2x + λ1 = 0,
3y 2 + λ2 = 0,
λ1 ≥ 0,
λ2 ≥ 0,
λ1 (x − a) = 0,
λ2 (y − 1) = 0.

23
En combinant les deux premières équations, on a 2x = −3y 2 . En utilisant cette équation, on peut éliminer x
de la première équation et obtenir λ1 = 6y 2 . Ainsi, on a λ2 = −3y 2 . Puisque λ1 ≥ 0 et λ2 ≥ 0, cela implique
que y = 0, ce qui n’est pas satisfaisable.
Par conséquent, le problème n’a pas de solution optimale sur R avec y ≥ 1.

c) La fonction objectif est convexe et les contraintes sont des inégalités. Les conditions KKT sont les suivantes :

3x2 + λ1 = 0,
1 + λ2 = 0,
λ1 ≥ 0,
λ2 ≥ 0,
λ1 (x − a) = 0,
λ2 (y − 1) = 0.

En utilisant la deuxième équation, on obtient λ2 = −1. En utilisant la première équation, on obtient λ1 =


−3x2 . Puisque λ1 ≥ 0, cela implique que x = 0. En utilisant à nouveau la première équation, on obtient
λ1 = 0. Ainsi, on a λ2 = −1.
Les conditions KKT impliquent que y = 1 et x = 0. On vérifie facilement que (x, y) = (0, 1) est un minimum
global de la fonction objective. Par conséquent, la solution optimale est (x, y) = (0, 1) avec une valeur
objective minimale de 1.

Exercice XIV
f (x) = x + x2
Déterminons ∂f (x) = |x| + x2 .
On a f est convexe comme somme de deux fonctions convexes.
∂f (x0( ) = {∇f (x0 )}
−x si x ≤ 0,
|x| =
x si x > 0.
∂f (0) = [∂fg (0); ∂fd (0)]
= [−1; 1]
(
−x + x2 si x ≤ 0,
f (x) =
x + x2 si x > 0.
(
−1 + 2x si x ≤ 0,
f ′ (x) = .
1 + 2x si x > 0.
On a donc :

-Si x ≤ 0
fg (x) = fd (x) = 2x − 1
∂f (x) = {2x − 1}

-Si x > 0
fg (x) = fd (x) = 2x + 1
∂f (x) = {2x + 1}

-Si x = 0 , fg′ (0) = −1 et fd′ (0) = 1


∂f (0) = [−1, 1]

24
En résumé, on a :

2x + 1
 si x > 0,
∂f (x) = [−1, 1] si x = 0,
si x < 0.

2x − 1

25

Vous aimerez peut-être aussi